[Rozgrzewka OM][MIX][Nierówności] Nierówności

Zadania z kółek matematycznych lub obozów przygotowujących do OM. Problemy z minionych olimpiad i konkursów matematycznych.
Regulamin forum
Wszystkie tematy znajdujące się w tym dziale powinny być tagowane tj. posiadać przedrostek postaci [Nierówności], [Planimetria], itp.. Temat może posiadać wiele różnych tagów. Nazwa tematu nie może składać się z samych tagów.
Awatar użytkownika
Slup
Użytkownik
Użytkownik
Posty: 795
Rejestracja: 27 maja 2016, o 20:49
Płeć: Kobieta
Lokalizacja: Warszawa
Podziękował: 23 razy
Pomógł: 156 razy

[Rozgrzewka OM][MIX][Nierówności] Nierówności

Post autor: Slup »

To chyba faktycznie na rozruszanie, bo jestem frajerem z nierówności i umiem ją zrobić.

Mamy:

\(\displaystyle{ 1 = \sum_{i=1}^n\frac{x_i}{\sum_{j=1}^nx_j} < \sum_{i=1}^n\frac{x_i}{x_i+x_{i+1}}}\)

(na pierwszy rzut oka powinno być tylko \(\displaystyle{ \leq}\), ale w rzeczywistości z warunku \(\displaystyle{ n>2}\) oraz \(\displaystyle{ 0<x_i+x_{i+1}}\) dla każdego \(\displaystyle{ i}\) wynika, że jest \(\displaystyle{ <}\)). To daje pierwszą nierówność. Dla dowodu drugiej wystarczy zauważyć dwa fakty. Po pierwsze można tak samo jak wyżej dowieść, że

\(\displaystyle{ 1 < \sum_{i=1}^n\frac{x_{i+1}}{x_i+x_{i+1}}}\)

Po drugie

\(\displaystyle{ n = \left(\sum_{i=1}^n \frac{x_i}{x_i+x_{i+1}}\right) + \left(\sum_{i=1}^n\frac{x_{i+1}}{x_i+x_{i+1}}\right)}\)

Zatem ostatecznie

\(\displaystyle{ \sum_{i=1}^n \frac{x_i}{x_i+x_{i+1}} = n - \sum_{i=1}^n\frac{x_{i+1}}{x_i+x_{i+1}} < n-1}\)

Jak już wspomniałem, nie znam się na nierównościach olimpijskich, więc proszę by ktoś z użytkowników, którzy są tutaj aktywni, umieścił kolejne zadanie.
Awatar użytkownika
Premislav
Użytkownik
Użytkownik
Posty: 15688
Rejestracja: 17 sie 2012, o 13:12
Płeć: Mężczyzna
Lokalizacja: Warszawa
Podziękował: 196 razy
Pomógł: 5221 razy

Re: [Rozgrzewka OM][MIX][Nierówności] Nierówności

Post autor: Premislav »

O to właśnie chodziło (nie znam innego rozwiązania). To może trudniejsze:
niech \(\displaystyle{ x,y,z\in \RR}\) spełniają warunek \(\displaystyle{ x^4+y^4+z^4+xyz=4}\). Proszę wykazać, że
\(\displaystyle{ x\le 2}\) oraz \(\displaystyle{ \sqrt{2-x}\ge \frac{y+z}{2}}\)
Awatar użytkownika
WolfusA
Użytkownik
Użytkownik
Posty: 208
Rejestracja: 27 sty 2017, o 19:43
Płeć: Mężczyzna
Podziękował: 17 razy
Pomógł: 9 razy

[Rozgrzewka OM][MIX][Nierówności] Nierówności

Post autor: WolfusA »

bardzo niezobowiązująco:    
Ostatnio zmieniony 27 kwie 2019, o 21:23 przez Jan Kraszewski, łącznie zmieniany 1 raz.
Powód: Niepoprawne tagowanie.
bosa_Nike
Użytkownik
Użytkownik
Posty: 1668
Rejestracja: 16 cze 2006, o 15:40
Płeć: Kobieta
Podziękował: 71 razy
Pomógł: 447 razy

[Rozgrzewka OM][MIX][Nierówności] Nierówności

Post autor: bosa_Nike »

Ukryta treść:    
Dla \(\displaystyle{ 4c>3b>2a>0}\) udowodnij

\(\displaystyle{ 2\left(\frac{a}{b}+\frac{b}{c}+\frac{c}{a}\right)-\left(\frac{b}{a}+\frac{c}{b}+\frac{a}{c}\right)\ge 3}\)
Awatar użytkownika
Premislav
Użytkownik
Użytkownik
Posty: 15688
Rejestracja: 17 sie 2012, o 13:12
Płeć: Mężczyzna
Lokalizacja: Warszawa
Podziękował: 196 razy
Pomógł: 5221 razy

Re: [Rozgrzewka OM][MIX][Nierówności] Nierówności

Post autor: Premislav »

Ukryta treść:    
Jak macie jakiś mniej rachunkowy dowód, to tradycyjnie zachęcam do podzielenia się nim.
Nowe:
niech \(\displaystyle{ x_1, x_2, \ldots x_n\ge 0}\).
Proszę udowodnić, że
\(\displaystyle{ \sum_{i=1}^{n} \frac{x_i}{1+x_1^2+\ldots+x_i^2}<\sqrt{n}}\)
Awatar użytkownika
Premislav
Użytkownik
Użytkownik
Posty: 15688
Rejestracja: 17 sie 2012, o 13:12
Płeć: Mężczyzna
Lokalizacja: Warszawa
Podziękował: 196 razy
Pomógł: 5221 razy

Re: [Rozgrzewka OM][MIX][Nierówności] Nierówności

Post autor: Premislav »

Znowu stanęło, mimo że zadanie proste (a może właśnie dlatego nikomu się nie chciało pisać; jak teraz patrzę, to nie jestem też pewien, czy nie widziałem go już kiedyś w tym dziale ).
Ukryta treść:    
Nowe zadanie (mam nadzieję, że jeszcze nie było);
dana jest liczba całkowita dodatnia \(\displaystyle{ k}\). Proszę wyznaczyć wszystkie takie liczby całkowite dodatnie \(\displaystyle{ n}\), że dla dowolnych liczb rzeczywistych nieujemnych \(\displaystyle{ x_1, \ldots x_n}\) spełniających warunek \(\displaystyle{ x_1+x_2+\ldots+x_n=n}\) zachodzi nierówność
\(\displaystyle{ kn+ \sum_{i=1}^{n}x_i^3\le n+k \sum_{i=1}^{n}x_i^2}\)
Awatar użytkownika
WolfusA
Użytkownik
Użytkownik
Posty: 208
Rejestracja: 27 sty 2017, o 19:43
Płeć: Mężczyzna
Podziękował: 17 razy
Pomógł: 9 razy

Re: [Rozgrzewka OM][MIX][Nierówności] Nierówności

Post autor: WolfusA »

Ukryta treść:    
Gdy ktoś się zgodzi (żeby nie było samowładzy) wrzucę kolejny numer.
Awatar użytkownika
Premislav
Użytkownik
Użytkownik
Posty: 15688
Rejestracja: 17 sie 2012, o 13:12
Płeć: Mężczyzna
Lokalizacja: Warszawa
Podziękował: 196 razy
Pomógł: 5221 razy

Re: [Rozgrzewka OM][MIX][Nierówności] Nierówności

Post autor: Premislav »

Bardzo elegancko! To jest zadanie ze Zwardonia 2004, pierwszy mecz matematyczny, zadanie 5.

Kod: Zaznacz cały

https://om.mimuw.edu.pl/static/app_main/camps/zwardon2004r.pdf


Zadajesz.
Awatar użytkownika
WolfusA
Użytkownik
Użytkownik
Posty: 208
Rejestracja: 27 sty 2017, o 19:43
Płeć: Mężczyzna
Podziękował: 17 razy
Pomógł: 9 razy

Re: [Rozgrzewka OM][MIX][Nierówności] Nierówności

Post autor: WolfusA »

Dana jest liczba \(\displaystyle{ n\in Z_+}\) zaś dowolne liczby \(\displaystyle{ x_1, \ldots, x_n, y_1, \ldots, y_n\in R_+}\) spełniają warunek \(\displaystyle{ x_1+\ldots+x_n=y_1+\ldots+y_n=1}\). Wykaż, że
\(\displaystyle{ |x_1-y_1|+\ldots+|x_n-y_n|\leq 2-\underset{1\leq i\leq n}{\min} \;\dfrac{x_i}{y_i}-\underset{1\leq i\leq n}{\min} \;\dfrac{y_i}{x_i}}\)
Awatar użytkownika
WolfusA
Użytkownik
Użytkownik
Posty: 208
Rejestracja: 27 sty 2017, o 19:43
Płeć: Mężczyzna
Podziękował: 17 razy
Pomógł: 9 razy

Re: [Rozgrzewka OM][MIX][Nierówności] Nierówności

Post autor: WolfusA »

Zadanie pochodzi z Junior Balkan MO 2014 Shortlist - zaadanie A9. Na innym znanym forum nie ma rozwiązania. Być może komuś uda się znaleźć w jakimś innym miejscu to zadanie z rozwiązaniem (mi się nie udało). Jeśli nie, to za dwa dni zmieniam zadanie.-- 4 cze 2019, o 12:27 --\(\displaystyle{ n\in Z_+ \wedge n\ge2\wedge a_1,a_2,...,a_n\in R}\). Jeśli zachodzi nierówność \(\displaystyle{ b<\frac{1}{n-1}\cdot\left(\sum_{i=1}^n a_i\right)^2-\sum_{i=1}^n a_i^2}\), to wykaż, że dla każdych całkowitych \(\displaystyle{ 1\le k<l\le n}\) mamy \(\displaystyle{ b<2a_ka_l}\).
Awatar użytkownika
Premislav
Użytkownik
Użytkownik
Posty: 15688
Rejestracja: 17 sie 2012, o 13:12
Płeć: Mężczyzna
Lokalizacja: Warszawa
Podziękował: 196 razy
Pomógł: 5221 razy

Re: [Rozgrzewka OM][MIX][Nierówności] Nierówności

Post autor: Premislav »

Może spróbuję odblokować ten wątek, nie wiem, czy mi się uda…
Ukryta treść:    
Miło byłoby, gdyby ktoś to sprawdził.
WolfusA, masz może do tego jakieś fajne rozwiązanie?
Awatar użytkownika
WolfusA
Użytkownik
Użytkownik
Posty: 208
Rejestracja: 27 sty 2017, o 19:43
Płeć: Mężczyzna
Podziękował: 17 razy
Pomógł: 9 razy

[Rozgrzewka OM][MIX][Nierówności] Nierówności

Post autor: WolfusA »

Jeszcze nie sprawdziłem Premislava, ale
Moje fajne rozwiązanie:    
Awatar użytkownika
Premislav
Użytkownik
Użytkownik
Posty: 15688
Rejestracja: 17 sie 2012, o 13:12
Płeć: Mężczyzna
Lokalizacja: Warszawa
Podziękował: 196 razy
Pomógł: 5221 razy

Re: [Rozgrzewka OM][MIX][Nierówności] Nierówności

Post autor: Premislav »

Że też ja na to nie wpadłem. Szkoda słów…

-- 10 lip 2019, o 00:34 --

ciach
Ostatnio zmieniony 10 lip 2019, o 15:22 przez Zahion, łącznie zmieniany 1 raz.
Powód: Poprawa wiadomości.
Awatar użytkownika
WolfusA
Użytkownik
Użytkownik
Posty: 208
Rejestracja: 27 sty 2017, o 19:43
Płeć: Mężczyzna
Podziękował: 17 razy
Pomógł: 9 razy

Re: [Rozgrzewka OM][MIX][Nierówności] Nierówności

Post autor: WolfusA »

Chcesz wrzucić nową, czy dalej moja kolej?
To zadanie to Putnam 1977 B5 (z 6 zadań) więc było ocenione na bardzo trudne przez organizatorów. Technicznie wymagające nie jest, ale znaleźć właściwą drogę już niełatwo.
Awatar użytkownika
Premislav
Użytkownik
Użytkownik
Posty: 15688
Rejestracja: 17 sie 2012, o 13:12
Płeć: Mężczyzna
Lokalizacja: Warszawa
Podziękował: 196 razy
Pomógł: 5221 razy

Re: [Rozgrzewka OM][MIX][Nierówności] Nierówności

Post autor: Premislav »

Dalej Twoja kolej, nie jestem usatysfakcjonowany tym, co tu odwaliłem.

BTW Fajnie byłoby zobaczyć dowód tej poprzedniej nierówności z shortlisty JBMO , ale to już może nad nią sobie posiedzę.
ODPOWIEDZ